3
$\begingroup$

Cross-posting this question from MSE.

Let $C\subseteq \mathbb R$ denote the Cantor set and let $A\subseteq C$ be dense with $|A| = |\mathbb R|$, and assume $A$ doesn't contain its infimum. Is there a locally compact, second countable, Hausdorff topology on $A$ that is finer than the right order topology (the topology with base $\{(a,\infty)\cap A: a\in A\}$)? If it helps, the continuum hypothesis can be assumed.

An equivalent formulation of the problem is to find a compact metric on $A\cup \{0\}$ refining the right order topology by identifying the point 0 with the point added in the 1-point compactification.

If $|C\setminus A|$ is finite, the result is easy as $C$ is compact in the Euclidean topology. If $C\setminus A$ is countable, then the same methods as this answer should hold. The difficulty is if $|C\setminus A| = |\mathbb R|$.

This question is a special case of this question.

$\endgroup$

1 Answer 1

5
$\begingroup$

This is not always possible (even under extra assumptions such as the continuum hypothesis). In particular, $\def\ZFC{\mathsf{ZFC}}\ZFC$ proves the following:

Proposition. If $A \subseteq C$ is a Bernstein set, then there is no complete metric on $A$ refining the right order topology.

Proof. We'll prove a stronger contrapositive: If there is a complete metric on an uncountable set $A$ refining the right order topology, then there is an uncountable set $F \subseteq A$ that is closed in the Euclidean topology.

Assume that $d$ is a complete metric on $A$. Since $d$ refines the right order topology, we have that the inclusion map $\iota : (A,d) \to (C,|\cdot|)$ is lower semicontinuous and so in particular Borel-measurable. This implies that $\iota$ as a subset of $(A,d) \times (C,|\cdot|)$ is Borel. Since $A$ is uncountable and since Borel sets have the perfect set property, there is a perfect set $G \subseteq \iota$. Let $F$ be the projection of $G$ to $C$. Note that since $\iota$ is a literal inclusion map, we just have that $G = \{(x,x) : x \in F\}$. In particular $F$ is an uncountable subset of $A$ that is closed (in both topologies).

Since Bernstein sets have no uncountable closed subsets, this implies that no such $d$ can exist if $A$ is a Bernstein set. $\square$

Finally, note that $\ZFC$ proves the existence of Bernstein set with the same cardinality as $\mathbb{R}$ without any additional set-theoretic assumptions.

$\endgroup$
3
  • $\begingroup$ Why must $F$ be closed in $C$? Projections of closed sets need not be closed. If you consider the embedding map $\iota:\mathbb Q^c\to \mathbb R$ (Euclidean on domain and codomain), then $\{(x,x):x\in \mathbb Q^c\cap[0,1]\}\subseteq \iota$ is closed in $\mathbb Q^c\times \mathbb R$, but $\mathbb Q^c\cap[0,1]$ is not closed in $\mathbb R$. It is closed in the subspace topology, but your argument seems to require $F$ to be closed in the topology on $C$, not on the subspace topology on $A$. $\endgroup$ Commented May 12 at 1:23
  • 1
    $\begingroup$ @daRoyalCacti $G$ can be taken to be compact, which is enough to ensure that it's closed in both topologies. $\endgroup$ Commented May 12 at 2:06
  • 1
    $\begingroup$ Compact will do it. Thank you $\endgroup$ Commented May 12 at 2:10

You must log in to answer this question.

Start asking to get answers

Find the answer to your question by asking.

Ask question

Explore related questions

See similar questions with these tags.